You are on page 1of 9

PHY4604 Exam 2 Solutions

PHY 4604 Exam 2 Solutions


(Total Points = 100)

Problem 1 (20 points)

Problem 1a (2 points): The wave function for an electron in a state with zero angular
momentum: (circle the correct answer)
(a) is zero everywhere (b) is spherically symmetric (c) depends on the angle from the z axis
(d) depends on the angle from the x axis
(e) is spherically symmetric for some shells and depends on the angle from the z axis for others

Problem 1b (2 points): The magnitude of the orbital angular momentum of an electron in an


atom is what multiple of h ? (l = 0, 1, 2, …) (circle the correct answer)
(a) 1 (b) ½ (c) l (l + 1) (d) 2l + 1 (e) l 2

Problem 1c (2 points): An electron is in a quantum state for which the magnitude of the orbital
momentum is 6 2h . How many allowed values of the z-component of the angular momentum
are there? (circle the correct answer)
(a) 4 (b) 5 (c) 7 (d) 8 (e) 9
I goofed up this problem. The correct answer is 17.

Problem 1d (2 points): If the wave function ψ is spherically symmetric then the radial
probability density is given by: (circle the correct answer)
(a) 4πr 2ψ (b) | ψ |2 (c) 4πr 2 | ψ |2 (d) 4π | ψ |2 (e) 4πr | ψ |2

Problem 1e (3 points): An electron in an atom is in a state with l = 3 and ml = 2. The angle


r
between L and the z-axis is given by: (circle the correct answer)
(a) 48.2º (b) 60º (c) 30º (d) 35.3º (e) 54.7º

Problem 1f (3 points): SU(2) is the group of 2×2 matrices, U, where:


(a) U ↑U = UU ↑ = 1 (b) U = U ↑ (c) det(U)=1 (d) U −1 = U ↑ (e) U * = U T
(Note: circle all the correct answers)

Problem 1g (3 points): If H is an Hermitian operator then:


(a) H ↑ H = HH ↑ = 1 (b) H = H ↑ (c) <H> ≥ 0 (c) <H2> ≥ 0
(e) its eigenvalues are real
(Note: circle all the correct answers)
v
Problem 1h (3 points): If L is the orbital angular momentum operator then:
(a) ∆Lx ∆Ly = 0 (b) ∆Lx ∆Ly ≥ h2 |< Lz >| (c) ∆Lx ∆Ly ≥ h2 (d) ∆Lz ∆L2 ≥ 0
(Note: circle all the correct answers)

Department of Physics Page 1 of 9


PHY4604 Exam 2 Solutions

Problem 2 (30 points): Consider a spin ½ system described by the Hamiltonian:


ε − iε 1 
H =  0 
 iε 1 ε 0 
where ε0 and ε1 are real positive constants.
(a) (6 points) Find the energy levels of the sustem. How many energy levels are there? What is
the ground state energy, E0, and the first excited state energy, E1?
Answer: There are two energy levels: E0 = ε0 – ε1, E1 = ε0 + ε1.
Solution: The energy levels are the solution of
ε 0 − λ − iε 1
=0
iε 1 ε0 − λ
which yields (ε 0 − λ ) 2 − ε12 = 0 , which implies that ε 0 − λ = ±ε1 and hence λ = ε 0 ± ε1 . There are
two energy levels, E0 = ε0 – ε1 and E1 = ε0 + ε1.
(b) (6 points) What are the (normalized) eigenkets corresponding to the ground state, |E0>, and
the first excited state , |E1>?
Answer:
1 i 1 − i
| E0 >=   | E1 >=  .
2 1 2  1 
Solution: For the ground state we see that
 ε 0 − iε1  a   ε 0 a − iε 1b  a
   =   = (ε 0 − ε 1 ) 
 iε1 ε 0  b   iε1a + ε 0b  b
and hence ε0a –iε1b = (ε0-ε1)a, which implies that a = ib and hence if b = 1 then a = i and
1 i
| E0 >=  .
2 1
For the first excited state we see that
 ε 0 − iε1  a   ε 0 a − iε1b  a
   =   = (ε 0 + ε1 ) 
 iε1 ε 0  b   iε1a + ε 0b  b
and hence ε0a –iε1b = (ε0+ε1)a, which implies that a = -ib and hence if b = 1 then a = -i and
1 − i
| E1 >=  .
2  1 

(c) (6 points) Now suppose that at t = 0 the system is in the state


1
| ψ 0 >=   .
 0
If you measure the energy of the state |ψ0>, what are the possible values you might get, and what
is the probability of getting each of them? What is the expectation value of the energy for this
state (i.e. the average energy <E>)?
Answer: You get energy E0 with probability P0 = ½ and you get energy E1 with probability P1 =
½ and the average energy is <E> = ε0.
Solution: We see that

Department of Physics Page 2 of 9


PHY4604 Exam 2 Solutions

1 1 − i 1
c0 =< E0 | ψ 0 >=
(− i 1)  = and P0 =| c0 |2 =
2  0 2 2
1 1 i 1
c1 =< E1 | ψ 0 >= (i 1)  = and P1 =| c1 |2 =
2  0 2 2
Thus, we get energy E0 with probability P0 = ½ and you get energy E1 with probability P1 = ½
and
< E >= P0 E0 + P1E1 = 12 (ε 0 − ε1 ) + 12 (ε 0 + ε1 ) = ε 0 .
Another way to get the average energy of the state |ψ0> is as follows
ε − i ε 1  1  ε 
< E >= (1 0) 0   = (1 0) 0  = ε 0 .
 iε 1 ε 0  0  0
(d) (6 points) What is <E2> for the state |ψ0> from part (c)? What is the uncertainty in the
energy of the state |ψ0> (i.e. what is ∆E = σE for this state)?
Answer: < E 2 >= ε 02 + ε12 , ∆E = < E 2 > − < E > 2 = ε1
Solution: We can calculate <E2> in two ways. It is equal to
< E 2 >= P0 E02 + P1E12 = 12 (ε 0 − ε1 ) 2 + 12 (ε 0 + ε1 ) 2 = ε 02 + ε12
Also,
ε − iε1  ε 0 − iε1  1  ε − iε1  ε 0   ε 2 + ε12 
< E 2 >= (1 0) 0    = (1 0) 0   = (1 0) 0  = ε 02 + ε12

i
 1ε ε 0  1 i ε ε 0  0 i
 1ε ε i
0  1  ε  2iε 0ε1 
Hence,
∆E = < E 2 > − < E > 2 = ε 02 + ε12 − ε 02 = ε1 .
(e) (6 points) If at t = 0 the system is in the state
1
| ψ 0 >=   ,
 0
what is |ψ(t)> at later time t?
Answer:
 cos(ε1t / h ) 
| ψ (t ) >= e − iε 0 t / h   .
 sin(ε1t / h ) 
Solution: We know that
− i  i  − i (ε 0 − ε 1 ) t / h i  − i  − i ( ε 0 + ε 1 ) t / h
| ψ (t ) >= c0 | E0 > e − iE 0 t / h + c1 | E1 > e − iE1t / h =  e +  e
2 1 2 1 
1 − iε 0 t / h  e + iε 1t / h + e − iε 1t / h  − iε 0 t / h  cos(ε1t / h) 
= e    
2  − ie + iε 1t / h + ie − iε 1t / h  = e sin(ε t / h )
   1 

Department of Physics Page 3 of 9


PHY4604 Exam 2 Solutions

Problem 3 (25 points): Consider the (one dimensional) wave function at t = 0 given by
ψ ( x ) = Ae − a | x | ,
ρ(x)
where A and a are real constants, and where |x| = x for x > 0 and
|x| = -x for x < 0.
(a) (4 points) Find the normalization constant A such that
+∞

∫ | ψ ( x) | dx = 1
2

−∞

and sketch the probability density ρ ( x) =| ψ ( x) |2 .


Answer: A = a -5.0 0.0 5.0

Solution: We see that


+∞ +∞ +∞ +∞
2 A2e −2 ax A2
∫ | ψ ( x) | dx = A ∫e ∫e
− 2 a| x| − 2 ax
2 2
dx = 2 A 2
dx = = =1,
−∞ −∞ 0
− 2a 0
a
and hence A = a . Thus,
ρ ( x) =| ψ ( x) |2 = ae −2 a| x|
(b) (6 points) Compute <x>, <x2>, and ∆x using the position space wave function ψ(x).
1 1
Answer: < x >= 0 , < x 2 >= 2 , and ∆x = .
2a 2a
Solution: We see that
+∞ +∞

∫ x |ψ ( x) | dx = A ∫ xe dx = 0
− 2 a| x|
< x >= 2 2

−∞ −∞
+∞ +∞ +∞
2 1
∫x | ψ ( x) | dx = A ∫x e dx = 2a ∫ x 2e − 2 ax dx = 2a
2 − 2 a| x|
< x >=
2 2 2 2
3
= 2,
−∞ −∞ 0
(2a ) 2a
where I used
+∞
2
∫x e
2 −bx
dx = .
0
b3
We see that
1
∆x = < x 2 > − < x > 2 = .
2a
(c) (7 points). Find the momentum space wave function at t =
0, where ρ(px)
+∞
1
φ ( px ) =
2πh − ∞ ∫ ψ ( x)e − ixp x / h dx

and verify that it is properly normalized. Sketch the probability


density ρ ( px ) =| φ ( px ) |2 .
Answer:
2ha ha 2 (ha )3
φ( px ) = , ρ ( p ) =| φ ( p ) | 2
=
-5.0 0.0 5.0

π p x2 + (ha) 2
x x
π ( px2 + (ha) 2 ) 2
Solution: We see that

Department of Physics Page 4 of 9


PHY4604 Exam 2 Solutions

+∞ +∞ 0
A A A
φ ( px ) = ∫e ∫e ∫e
− ixp x / h − a | x | − ixp x / h − ax − ixp x / h + ax
e dx = e dx + e dx
2πh −∞ 2πh 0 2πh −∞
+∞ +∞
A A a  1 1 
∫ e x dx + ∫ e x dx =  
− ( ip / h + a ) x − ( − ip / h + a ) x
= +
2πh 0 2πh 0 2πh  (ip x / h + a ) (−ip x / h + a) 
a 2a a 2a 2ha ha
= = =
2πh (ip x / h + a )(−ip x / h + a ) 2πh ( px / h ) + a
2 2
π p + (ha ) 2
2
x
Thus,
2 (ha )3
ρ ( px ) =| φ ( px ) |2 =
π ( px2 + (ha) 2 ) 2
and we see that
+∞ +∞
2 (ha)3 2(ha)3 π
∫ | φ ( px ) | dpx = π −∫∞ ( px2 + (ha) 2 ) 2 x
= =1,
2
dp
−∞
π 2(ha)3
where I used
+∞
1 π
∫ (p
0
2
x +b )
2 2
dpx = 3 .
4b
2
(d) (6 points). Compute <px>, <px >, and ∆px using the momentum space wave function φ(px).
Answer: < px >= 0 , < p x2 >= (ha) 2 , and ∆p x = ha .
Solution: We see that
+∞ +∞
2(ha)3 px
< px >= ∫ px | φ ( px ) |2 dpx = ∫ dp = 0
−∞
π − ∞ ( px + (ha) 2 ) 2 x
2

+∞ +∞
2( ha ) 3 p x2
< p x2 >= ∫ px | φ ( px ) | dpx = ∫ ( px2 + (ha)2 )2 dpx
2 2

−∞
π −∞

2( ha ) 3
π
= = ( ha ) 2
π 2( ha )
where I used

p x2 π
∫0 ( px2 + b 2 )2 dpx = 4b .
We see that
∆p x = < p x2 > − < p x > 2 = ha
(e) (2 points). What is ∆x ∆px? Is it consistent with the uncertainty principle?
h h
Answer: ∆x∆px = 2 >
2 2
Solution: We see that
1 1 2
∆x∆px = ha = h= h.
2a 2 2

Department of Physics Page 5 of 9


PHY4604 Exam 2 Solutions

Problem 4 (25 points): Suppose we have two vector operators


r r r r
( J1 )op and ( J 2 )op with [( J1 )op , ( J 2 )op ] = 0
and each of the vectors obey the same SU(2) “lie algebra”:
[( J1i )op , ( J1 j )op ] = iε ijk ( J1k )op and [( J 2i )op , ( J 2 j )op ] = iε ijk ( J 2 k )op .
The states |j1m1> are the eigenkets of ( J12 ) op and ( J1z )op and the states |j2m2> are the eigenkets
of ( J 22 ) op and ( J 2 z )op as follows:

( J12 )op | j1m1 >= j1 ( j1 + 1) | j1m1 > ( J 22 )op | j2 m2 >= j2 ( j2 + 1) | j2 m2 >


( J1z )op | j1m1 >= m1 | j1m1 > ( J 2 z )op | j2 m2 >= m2 | j2 m2 >
Also we know that
( J1± )op | j1m1 >= j1 ( j1 + 1) − m1 (m1 ± 1) | j1m1 ± 1 >
( J 2± )op | j2 m2 >= j2 ( j2 + 1) − m2 (m2 ± 1) | j2 m2 ± 1 >
± ±
where ( J1 )op = ( J1x )op ± i ( J1 y )op and ( J 2 )op = ( J 2 x )op ± i ( J 2 y )op . Now consider the vector
sum of the two operators,
r r r
( J )op = ( J1 )op + ( J 2 )op or ( J i )op = ( J i1 )op + ( J i 2 )op for i = 1,2, 3.
(a) (5 points): Show that
r r r r r r r r
( J 2 )op = ( J )op ⋅ ( J )op = ( J1 + J 2 )op ⋅ ( J1 + J 2 )op = ( J12 )op + ( J 22 )op + 2( J1 )op ⋅ ( J 2 )op
= ( J12 )op + ( J 22 )op + ( J1+ )op ( J 2− )op + ( J1− )op ( J 2+ )op + 2( J1z )op ( J 2 z )op
Solution: First we note that
J1x = 12 ( J1+ + J1− ) J 2 x = 12 ( J 2+ + J 2− )
and
J1 y = 1
2i
( J1+ − J1− ) J2y = 1
2i
( J 2+ − J 2− )
Hence,
r r
J 2 = J12 + J 22 + 2 J1 ⋅ J 2 = J12 + J 22 + 2 J1x J 2 x + 2 J1 y J 2 y + 2 J1z J 2 z
= J12 + J 22 + 12 ( J1+ + J1− )( J 2+ + J 2− ) − 12 ( J1+ − J1− )( J 2+ − J 2− ) + 2 J1z J 2 z
= J12 + J 22 + J1+ J 2− + J1− J 2+ + 2 J1z J 2 z
(b) (5 points): Evaluate the following in SU(2).
3×2= 4×3= 5×3=
5×4= 2×3×4=
Answers:
3 × 2 = 4 + 2 (j1 = 1, j2 = ½, j = 3/2, 1/2)
4 × 3 = 6 + 4 + 2 (j1 = 3/2, j2 = 1, j = 5/2, 3/2, 1/2)
5 × 3 = 7 + 5 + 3 (j1 = 2, j2 = 1, j = 3, 2, 1)
5 × 4 = 8 + 6 + 4 + 2 (j1 = 2, j2 = 3/2, j = 7/2, 5/2, 3/2, 1/2)
2 × 3 × 4 = 2 × (6 + 4 + 2) = 2×6 + 2×4 + 2×2 = 7 + 5 +5 + 3 + 3 + 1
where I used
2 × 6 = 7 + 5 (j1 = 1/2, j2 = 5/2, j = 3, 2)

Department of Physics Page 6 of 9


PHY4604 Exam 2 Solutions

2 × 4 = 5 + 3 (j1 = 1/2, j2 = 3/2, j = 2, 1)


2 × 2 = 3 + 1 (j1 = 1/2, j2 = 1/2, j = 1, 0)
(c) (15 points): Now consider the case where j1 = 1 and j2 = ½ (i.e. 3 × 2) and define the states as
follows:
| Y11 >1 =| 11 >
|↑>2 =| 12 12 >
| Y10 >1 =| 10 > and
|↓>2 =| 12 − 12 >
| Y1−1 >1 =| 1 − 1 >
Now consider the two superposition states
1 2 2 1
| + >≡ | Y11 >1|↓> 2 + | Y10 >1|↑> 2 and | − >≡ | Y11 >1|↓> 2 − | Y10 >1|↑> 2 .
3 3 3 3
Calculate the following and express your answer in terms of |+> and |->:
(1) ( J1 + J 2 ) | + > (2) ( J1 + J 2 ) | − >
2 2 2 2

+ − − + + − − +
(3) ( J1 J 2 + J1 J 2 + 2 J1z J 2 z ) | + > (4) ( J1 J 2 + J1 J 2 + 2 J1z J 2 z ) | − >
(5) J z | + > (6) J z | − > (7) J | + > (8) J | − >
2 2

Are the states | ± > eigenstates of the J2 and Jz and if so what are their eigenvalues?
Answer:
(1) ( J 1 + J 2 ) | + >= 114 | + > (2) ( J 1 + J 2 ) | − >= 114 | − >
2 2 2 2

+ − − +
(3) ( J1 J 2 + J1 J 2 + 2 J1z J 2 z ) | + >=| + >
+ − − +
(4) ( J1 J 2 + J1 J 2 + 2 J1z J 2 z ) | − >= −2 | − >
(5) J z | + >= 12 | + > (6) J z | − >= 12 | − >
(7) J | + >= 154 | + > (8) J | − >= 34 | − >
2 2

The state |+> correspomds to j = 3/2 m = ½ and |-> corresponds to j = ½ and m = ½.


Solution: We know that
J12 | Y11 >1 = J12 | 11 >= 1(1 + 1) | 11 >= 2 | Y11 >1 J 22 |↑> 2 = J 22 | 12 12 >= 12 ( 12 + 1) | 12 12 >= 34 |↑> 2
J12 | Y10 >1 = J12 | 10 >= 1(1 + 1) | 10 >= 2 | Y10 >1 J 22 |↓> 2 = J 22 | 12 − 12 >= 12 ( 12 + 1) | 12 − 12 >= 34 |↓> 2
J1z | Y11 >1 = J1z | 11 >= 1 | 11 >= 1 | Y11 >1 J 2 z |↑>2 = J 2 z | 12 12 >= 12 | 12 12 >= 12 |↑>2
J1z | Y11 >1 = J1z | 10 >= 0 | 10 >= 0 J 2 z |↓>2 = J 2 z | 12 − 12 >= − 12 | 12 − 12 >= − 12 |↓>2
J1+ | Y11 >1 = J1+ | 11 >= 0
J1+ | Y10 >1 = J1+ | 10 >= 1(1 + 1) + 0(0 + 1) | 11 >= 2 | Y11 >1

J1− | Y11 >1 = J1− | 11 >= 1(1 + 1) − 1(1 − 1) | 10 >= 2 | Y10 >1
J 2+ |↑> 2 = J 2+ | 12 12 >= 0
J 2+ |↓> 2 = J 2+ | 12 − 12 >= 1
2 ( 12 + 1) + 12 (− 12 + 1) | 12 12 >=|↑> 2
J 2− |↑>2 = J 2− | 12 12 >= 1
2 ( 12 + 1) − 12 ( 12 − 1) | 12 − 12 >=|↓>2
J 2− |↓>2 = J 2− | 12 − 12 >= 0
and hence

Department of Physics Page 7 of 9


PHY4604 Exam 2 Solutions

 1 2  1 2
J z | + >= ( J1z + J 2 z ) | Y11 >1|↓> 2 + | Y10 >1|↑> 2  = (1 − 12 ) | Y11 >1|↓> 2 +(0 + 12 ) | Y10 >1|↑> 2 = 12 | + >
 3 3  3 3

 2 1  2 1
J z | − >= ( J1z + J 2 z ) | Y11 >1|↓> 2 − | Y10 >1|↑> 2  = (1 − 12 ) | Y11 >1|↓> 2 −(1 − 12 ) | Y10 >1|↑> 2 = 12 | − >
 3 3  3 3

 1 2  1 2
J12 | + >= J12  | Y11 >1|↓> 2 + | Y10 >1|↑> 2  = (2) | Y11 >1|↓> 2 +(2) | Y10 >1|↑> 2 = 2 | + >
 3 3  3 3
 2 1  2 2
J12 | − >= J12  | Y11 >1|↓> 2 − | Y10 >1|↑> 2  = (2) | Y11 >1|↓> 2 −(2) | Y10 >1|↑> 2 = 2 | − >
 3 3  3 3
 1 2  1 2
J 22 | + >= J 22  | Y11 >1|↓> 2 + | Y10 >1|↑> 2  = ( 34 ) | Y11 >1|↓> 2 +( 34 ) | Y10 >1|↑> 2 = ( 34 ) | + >
 3 3  3 3
 2 1  2 2
J 22 | − >= J 22  | Y11 >1|↓> 2 − | Y10 >1|↑> 2  = ( 34 ) | Y11 >1|↓> 2 −( 34 ) | Y10 >1|↑> 2 = ( 34 ) | − >
 3 3  3 3
We also see that
 1 2 
2 J 1 z J 2 z | + >= 2 J 1 z J 2 z  | Y11 >1 |↓> 2 + | Y10 >1 |↑> 2 
 3 3 
1 2 1
= 2 (1)( − 12 ) | Y11 >1 |↓> 2 + 2( 0)( + 12 ) | Y10 >1 |↑> 2 = − | Y11 >1 |↓> 2
3 3 3

=−
1 1
3 3
( 1
|+ >+ 2|−> =− |+ >+ 2|−>
3
) ( )
and
 2 1 
2 J1z J 2 z | − >= 2 J1z J 2 z  | Y11 >1|↓> 2 − | Y10 >1|↑> 2 
 3 3 
2 1 2
= 2(1)(− 12 ) | Y11 >1|↓> 2 −2(0)(+ 12 ) | Y10 >1|↑> 2 = − | Y11 >1|↓> 2
3 3 3

=−
2 1
3 3
( 1
)
|+ > + 2 |− > = − 2|− > + 2 |+ >
3
( )
and hence,
2 J1z J 2 z | ± >= −
3
(
1 3 1
)
(( 2 m 2 ) | ± > + 2 | m > .
Also,

Department of Physics Page 8 of 9


PHY4604 Exam 2 Solutions

 1 2 
( J1+ J 2− + J1− J 2+ ) | + >= ( J1+ J 2− + J1− J 2+ ) | Y11 >1|↓> 2 + | Y10 >1|↑> 2 
 3 3 
1 2 1 2
= 2 | Y10 >1|↑> 2 + 2 | Y11 >1|↓> 2 = 2 | Y11 >1|↓> 2 + | Y10 >1|↑> 2
3 3 3 3
2
3
(
|+>+ 2|−> +
3
2
) ( 2 |+ > −|− > = ) 4
3
|+>+
3
2
|−>
and
 2 1 
( J1+ J 2− + J1− J 2+ ) | − >= ( J1+ J 2− + J1− J 2+ ) | Y11 >1|↓> 2 − | Y10 >1|↑> 2 
 3 3 
2 1 2 1
= 2 | Y10 >1|↑> 2 − 2 | Y11 >1|↓> 2 = − | Y11 >1|↓> 2 +2 | Y10 >1|↑> 2
3 3 3 3

=−
3
2
(
|+>+ 2|−> +
2
3
) ( 4
)
2 |+ > −|− > = − |− > +
3 3
2
|+>
Hence,
4 2
( J 1+ J 2− + J1− J 2+ ) | ± >= ± |±>+ | m >,
3 3
where I used

| Y11 >1|↓> 2 =
1
3
(
|+>+ 2|−> ) and | Y10 >1|↑> 2 =
1
3
( 2 |+ > −|− > . )
We see that
J 2 | ± >= ( J12 + J 22 + J1+ J 2− + J1− J 2+ + 2 J1z J 2 z ) | ± >
3 4
= 2|± > + |± > ± |± > +
4 3 3
2 1
(
| m > − (( 32 m 12 ) | ± > + 2 | m >
3
)
 3 4 1   2 2
=  2 + ± − ( 32 m 12 )  | ± > + − |m >

 4 3 3   3 3 
 3 4 1 1  27 18  9 6
=  2 + ± − ±  | ± >=  ±  | ± >=  ±  | ± >
 4 3 2 6  12 12  4 4
and hence
9 6 15 9 6 3
J 2 | + >==  +  | + >= | + > and J 2 | − >==  −  | − >= | − > .
4 4 4 4 4 4
Thus |+> correspomds to j = 3/2 m = ½ and |-> corresponds to j = ½ and m = ½.

Department of Physics Page 9 of 9

You might also like